LSAT and Law School Admissions Forum

Get expert LSAT preparation and law school admissions advice from PowerScore Test Preparation.

User avatar
 Dave Killoran
PowerScore Staff
  • PowerScore Staff
  • Posts: 5849
  • Joined: Mar 25, 2011
|
#44056
Complete Question Explanation
(The complete setup for this game can be found here: lsat/viewtopic.php?t=16041)

The correct answer choice is (E)

If M is the chairperson, then M and J are the two tenants, and because M cannot be selected with P, the three homeowners must be Q, R, and S. Thus, answer choice (E) is correct.

Note that even if you did not see this inference during your setup, answer choices (A) and (B) can be eliminated because they are both tenants (and J and M are already the two tenants on the committee), and answer choice (D) can be eliminated because of the sixth rule. Thus, even without the “J or M as chairperson” super-inference, you can eliminate three of the answer choices, leaving you with the ability to use a hypothetical to choose between (C) and (E).

Get the most out of your LSAT Prep Plus subscription.

Analyze and track your performance with our Testing and Analytics Package.